Property Learning Questions

¡Supera tus tareas y exámenes ahora con Quizwiz!

To buy a house, an investor secured a $10,000 mortgage from a bank. The bank promptly and properly recorded its mortgage. Subsequently, the investor financed certain improvements to the house with a $2,000 mortgage on the land from a finance company. The finance company promptly and properly recorded its mortgage. Before the investor made a payment on either mortgage, the federal government announced that it would begin storing nuclear waste products in the area. The value of property, including the investor's house, plummeted. The investor did not pay either the bank or the finance company, and the bank brought a proper action to foreclose, notifying both the investor and the finance company. A buyer bought the house at the foreclosure sale for $6,000, which was its fair market value. There are no special statutes in the jurisdiction regarding deficiency judgments. What does the investor owe?

$4,000 to the bank and $2,000 to the finance company. Absent any anti-deficiency statutes, the investor remains personally liable to pay for any shortfall arising from the foreclosure sale. Proceeds from the sale are used to satisfy the loan that was foreclosed first. Hence, all of the proceeds ($6,000) went to the bank. Thus, the investor must pay the balance still due the bank ($4,000) and the entire amount of the finance company's mortgage ($2,000), which is terminated by the foreclosure of the senior mortgage.

If A and B own property as joint tenants, and B dies leaving a will devising her interest in the property to C, who owns the property?

A only. A testamentary disposition by one joint tenant will not sever a joint tenancy. A will devising a joint tenant's interest to another is inoperative as to joint tenancy property because when the co-tenant who is the testator dies (which is when the will becomes effective), her rights in the joint tenancy property are extinguished, and the will has no effect on them. Thus, upon B's death the property is freed from her concurrent interest, leaving A the sole owner and C with no interest in the property.

At common law, a conveyance of property from O "to O and A as joint tenants with right of survivorship" creates a __________.

Tenancy in common. Three forms of concurrent ownership in land: joint tenancy with right of survivorship, tenancy in common, and tenancy by the entirety. In joint tenancy, each co-tenant owns an undivided share of the property, and the surviving co-tenant has the right to the whole estate (right of survivorship).At common law, a conveyance of property from O "to O and A as joint tenants with right of survivorship" creates a tenancy in common.A tenancy in common is a concurrent estate with no right of survivorship. A tenancy by the entirety is a marital estate akin to a joint tenancy in that four unities (plus a fifth-marriage) are required for its creation, and the surviving spouse has the right of survivorship. A conveyance from O "to O and A" does not satisfy the unities of time and title because O acquired his interest first by another instrument. Thus, the conveyance creates a tenancy in common rather than a joint tenancy. The conveyance does not create a tenancy for years at common law. A tenancy for years is a leasehold estate in land wherein the tenant has a present possessory interest in the leased premises and the landlord has a reversion. Here, O and A are not in a landlord-tenant relationship.

For purposes of determining title by adverse possession, when is tacking not available?

For purposes of determining title by adverse possession, tacking is not available when one adverse claimant ousts the other or the first claimant abandons and the next claimant goes into possession. Periods of adverse possession between two successive claimants may be tacked together to make up the full statutory period if there is privity of possession between the claimants. Privity is satisfied if the first adverse claimant purports to transfer the land to the next; i.e., the subsequent possessor takes by descent, by devise, or by deed purporting to convey title.

Statutory redemption is the right of a mortgagor to recover the land after the foreclosure sale has occurred, usually by paying __________.

the foreclosure sale price Statutory redemption is the right of a mortgagor to recover the land after the foreclosure sale has occurred, usually by paying the foreclosure sale price. About half the states provide a statutory right to redeem for some fixed period after the foreclosure sale has occurred, usually six months or one year. The amount to be paid is generally the foreclosure sale price, rather than the amount of the original debt. This right extends to mortgagors and, in some states, to junior lienors. In contrast, equitable redemption is the right of a mortgagor to recover the land by paying the amount overdue on the mortgage, plus interest, at any time before the foreclosure sale. If the mortgagor has defaulted and the mortgage or note contained an acceleration clause, then the full balance of the mortgage must be paid in order to redeem in equity.

A landlord entered into a 10-year lease of a building with an auctioneer, who planned to use the building itself for a storage area and the covered porch at the front of the building for auctions. A term in the auctioneer's lease stated, "Lessor agrees to maintain all structures on the property in good repair." Four years into the lease, the landlord sold the property to a buyer. The buyer did not agree to perform any obligations under the lease. As instructed, the auctioneer began paying rent to the buyer. In the fifth year of the lease, the porch roof began to leak. Citing the lease terms, the auctioneer asked the buyer to repair the roof. He continually refused to do so. The auctioneer finally repaired the roof herself at a cost of $2,000. The auctioneer then brought an appropriate lawsuit to recover the money. Absent any other facts, what is the auctioneer likely to recover?

$2,000 from either the buyer or the landlord, because they are both in privity with the auctioneer. A landlord's promise in a lease to maintain the property does not terminate because the property is sold. Although no longer in privity of estate, the original landlord and tenant remain in privity of contract, and the original landlord remains liable on the covenant unless there is a novation. A novation substitutes a new party for an original party to the contract. It requires the assent of all parties and completely releases the original party. Because neither the auctioneer nor the buyer has agreed to a novation, the landlord remains liable for the covenant because he and the auctioneer remain in privity of contract even after the sale. Thus, the promise to repair can be enforced against the landlord. When leased property is sold, the purchaser may be liable for his predecessor's promises if the promise runs with the land. A covenant in a lease runs with the land if the parties to the lease so intend and the covenant touches and concerns the land. Generally, promises to do a physical act, such as maintain or repair the property, are considered to run with the land. Thus, the buyer is liable because he is in privity of estate with the auctioneer and the covenant to repair runs with the land. Consequently, both the landlord and the buyer are potentially liable to the auctioneer for the repairs. While it is true that the sale/assignment to the buyer did not sever the landlord's obligation to the auctioneer, as explained above, the landlord is not the only person who is liable to the auctioneer.

Which of the following statements is correct regarding covenants against assignment or sublease?

If a landlord consents to one transfer that violates a covenant against assignment or sublease, he waives his right to avoid future transfers. This is the Rule in Dumpor's Case. The landlord may reserve the right to avoid future transfers, but such reservation must take place at the time of granting consent. A covenant against assignment or sublease is NOT an unreasonable restraint on alienation. All jurisdictions permit and enforce such covenants. A covenant against assignment does NOT prevent the tenant from subleasing her interest. Covenants against assignment or sublease are strictly construed against the landlord. Thus, a covenant prohibiting assignment does not prohibit subleasing and vice versa. If a tenant transfers her interest in violation of a covenant against assignment or sublease, the transfer is NOT void. However, the landlord usually may terminate the lease under the lease terms or a statute or sue for damages.

Which of the following is true when a seller of land breaches the implied covenant of marketable title?

The closing date may be extended to allow the seller time to cure Every land sale contract contains an implied covenant that the seller will provide marketable title at closing. Marketable title is title reasonably free from doubt, which a reasonably prudent buyer would accept. While it need not be perfect title, it must not present the buyer with an unreasonable risk of litigation. Generally, this means an unencumbered fee simple with good record title. If the buyer determines that the seller's title is unmarketable, he must notify the seller and give her a reasonable time to cure, even if this requires extending the closing date, and even if time is of the essence. When a seller of land breaches the implied covenant of marketable title, rescission IS available as a remedy. If the seller fails to cure the defects as explained above, then the buyer may rescind the contract, sue for damages for breach, get specific performance with abatement of the purchase price, or(in some jurisdictions) require the seller to quiet title. A court also may order rescission before the delivery date of an installment land contract if the buyer shows that the seller cannot possibly cure the defects in time. In contrast with the buyer's remedies, the seller CANNOT obtain specific performance or damages (unless the seller cures the title defect within a reasonable time). When a seller of land breaches the implied covenant of marketable title, the buyer may NOT sue for breach after closing. The implied covenant of marketable title applies at the contract stage of a land sale transaction, before the closing (i.e., exchange of purchase price and deed). The closing extinguishes the contract, which is said to merge with the deed. Then, absent fraud, the seller is no longer liable on this implied covenant; the buyer must rely on any assurances made in the deed.

When can a mortgagor exercise her statutory right of redemption?

after the foreclosure sale A mortgagor cannot exercise her statutory right of redemption before default, before the foreclosure sale or only during the foreclosure sale. In the states that provide it, the statutory right to redeem exists for a fixed period after the foreclosure sale. By contrast, all jurisdictions recognize the mortgagor's right of redemption in equity, which exists until the date of sale and is cut off by foreclosure. A mortgagor may purchase the land at the foreclosure sale, but a statutory right of redemption provides a grace period after foreclosure when the mortgagor may redeem the property.

To acquire a prescriptive easement on property, the claimant's use does not need to be __________.

exclusive. Acquiring a prescriptive easement is analogous to acquiring property by adverse possession, except that the use need not be exclusive (i.e., the user may share the use with the owner or other easement claimants). The use must be: 1. Open and notorious; 2. Adverse; and 3. Continuous and uninterrupted for the statutory period.

If an adverse possessor uses land in violation of a recorded real covenant for the limitations period, she:

Takes title free of the real covenant The nature of the title obtained through adverse possession depends on the occupier's activities on the land. If an adverse possessor uses the land in violation of a real covenant (i.e., a written promise to do or refrain from doing something on the land), she takes title free of the covenant EVEN IF she had knowledge of it. However, if she complies with the covenant for the statutory period, she takes title subject to the real covenant. In either case, if an adverse possessor uses land for the limitations period, she DOES take title to the land.

A developer owned a 30-acre tract of farmland. As required by law, the developer filed a plat with the county planning board, but did not record it. The plat divided the parcel into 87 one-third-acre residential lots. A one-acre strip on the eastern edge of the parcel that abutted a busy highway was set aside for commercial development. The plat restricted each lot to a single residence and banned all "nonconforming detracting structures or appurtenances," including "free-standing flagpoles more than six feet in height, television antennas and receiving equipment of excessive size and obtrusiveness, and windmills." The restrictive clause was put into the deeds of all the residential lots in the subdivision, except for the deeds to lots 23, 24, and 25. This oversight was due to an error by the developer's secretary. All the other lots had deeds stating that the restriction applied "to the grantee and his or her heirs and assigns." A homeowner purchased lot 24 and duly recorded her deed in the office of the county recorder of deeds. The developer's salesperson had orally informed the homeowner of the general restrictions applicable to lots in the subdivision. A year later, a sports bar purchased the one-acre commercial strip and installed a large satellite dish. Two years later, the homeowner sold her property to a buyer. The homeowner never mentioned any of the restrictions to the buyer. The buyer put a satellite dish on top of his house. His dish was not as large as the bar's dish, but it was obviously bigger than any of his neighbors' modest antennas. The owners of 15 lots in the subdivision sue the buyer, demanding that he remove the dish. If the court finds for the buyer, what is the likely reason?

The buyer is not charged with record notice based on other deeds given by a common grantor. When a developer subdivides land into several parcels and some of the deeds contain negative covenants but some do not, negative covenants or equitable servitudes binding all the parcels in the subdivision may be implied under the doctrine of "reciprocal negative servitudes." Two requirements must be met before reciprocal negative servitudes will be implied: (i) a common scheme for development, and (ii) notice of the covenants. The second requirement may be satisfied by actual notice, record notice, or inquiry notice. Here, the buyer has not been given actual notice, and the antenna restriction is not so obvious that the appearance of the neighborhood would provide the buyer with inquiry notice. Finally, the buyer has no record of the restriction in his chain of title to establish record notice. If the buyer had been the first purchaser of the lot, some courts might require him to read all deeds given by a common grantor, but the better view does not require such a search. In any case, the buyer's grantor here is the homeowner, and the restriction was not contained in her deed; the buyer thus does not have record notice of it and is not bound.

Which of the following parties cannot be protected as a bona fide purchaser of land?

A devisee of the land A devisee of the land cannot be protected as a bona fide purchaser ("BFP") of land. Notice and race-notice recording acts protect BFPs from prior unrecorded conveyances of the same property. A BFP is a purchaser who takes land without notice of a prior instrument and pays valuable consideration. Donees, heirs, and devisees are not BFPs because they do not give value for their interests; i.e., they are not purchasers. A mortgagee of the land can be protected as a BFP of land. Mortgagees for value (but not those who receive a mortgage only as security for a preexisting debt) are treated as purchasers, either expressly by recording acts or by judicial classification. Thus, mortgagees for value who take without notice can be protected as BFPs. A purchaser from an heir to the land can be protected as a BFP of land. Donees, heirs, and devisees themselves are not purchasers and thus cannot be BFPs. However, one who buys land from such a party will be protected against a prior unrecorded conveyance from the record owner. A donee from a bona fide purchaser of the land can be protected as a BFP of land. Under the shelter rule, anyone who takes from a BFP will be treated like a BFP (i.e., will prevail against any interest her transferor would have prevailed against). This rule exists to protect the BFP by preserving his ability to convey property. It applies even when his transferee had actual knowledge of a prior unrecorded interest or did not take for substantial pecuniary value (i.e., was a donee). However, a non-BFP who previously had title cannot acquire BFP status by selling the land to a BFP and then repurchasing it.

In which of the following situations must the tenant continue to pay a portion of the rent?

A paramount title holder takes possession of an unused barn on the leased premises and stores farm equipment in it. Every lease contains an implied covenant that neither the landlord nor someone with paramount title will interfere with the tenant's quiet enjoyment and possession of the premises. This covenant is breached by the tenant's total or partial actual eviction from the leased premises. Total actual eviction occurs when the landlord or a paramount title holder excludes the tenant from the entire leased premises. This terminates the tenant's obligation to pay rent. Partial actual eviction occurs when the tenant is excluded from only part of the leased premises. Partial eviction by the landlord relieves the tenant of the obligation to pay rent for the entire premises, even though the tenant continues in possession of the remainder of the premises. Partial eviction by a paramount title holder results in an apportionment of rent; i.e., the tenant is liable for the reasonable rental value of the portion that he continues to possess. A paramount title holder's taking possession of an unused barn constitutes partial actual eviction. Thus, rent will be apportioned.

Which of the following would render title to land unmarketable?

An existing violation of a zoning ordinance Every land sale contract contains an implied covenant that the seller will provide marketable title at closing. Marketable title is title reasonably free from doubt, which a reasonably prudent buyer would accept. While it need not be perfect title, it must not present the buyer with an unreasonable risk of litigation. Generally, this means an unencumbered fee simple with good record title. The mere existence of a zoning ordinance does not constitute an encumbrance. However, title to land that currently violates a zoning ordinance would be considered unmarketable. A mortgage that the seller is poised to satisfy at closing would not render title to land unmarketable. A seller has the right to satisfy a mortgage or lien at the closing with the proceeds from the sale. Thus, as long as the purchase price is sufficient and this is accomplished simultaneously with the transfer of title (e.g., through the use of escrows), the buyer cannot claim that title is unmarketable; the closing will result in a marketable title. A visible easement that benefits the property would not render title to land unmarketable. Most courts hold that a beneficial easement (e.g., a utility easement) that was visible or known to the buyer does not constitute an encumbrance. In contrast, an easement that reduces the value of the property or is unknown to the buyer constitutes an encumbrance that renders title unmarketable. A very slight encroachment onto an adjacent landowner's land would not render title to land unmarketable. Regardless of whether an adjacent landowner is encroaching on the seller's land or vice versa, an encroachment will not render title unmarketable if: 1. It is very slight (only a few inches) and does not inconvenience the owner on whose land it encroaches; 2. The owner encroached upon has indicated that he will not sue on it; or 3. It has existed for so long (many decades) that it has become legal by adverse possession (if the state recognizes adverse possession title as marketable). In contrast, a significant encroachment constitutes a title defect that renders title unmarketable.

A landowner conveyed her parcel of land to "my brother and my sister jointly, with right of survivorship." Shortly thereafter, the brother was in an automobile accident. The driver of the other vehicle sued the brother on a theory of negligence, and obtained a judgment in the amount of $250,000. Because the brother did not have insurance or enough cash to satisfy the judgment, the driver levied on the brother's interest in the land. What interest will the driver most likely take?

An undivided one-half interest, regardless of whether the brother and the sister's title to the land is construed as a joint tenancy or a tenancy in common. A joint tenancy is a concurrent estate with a right of survivorship, while a tenancy in common does not have a right of survivorship. At common law, the conveyance here would qualify as a joint tenancy because the unities of time, title, interest, and possession are present in the conveyance. Although under modern law a joint tenancy must be created with specific language or else it will be presumed to be a tenancy in common, the conveyance here still would probably qualify as a joint tenancy, even though it did not use the words "joint tenancy," because it contained the "right of survivorship" language. However, regardless of whether the estate is characterized as a joint tenancy or a tenancy in common, one tenant's interest may be transferred without the consent of the other tenant, and a creditor may levy on the interest. In most jurisdictions, a lien against one joint tenant's interest does not sever the joint tenancy until the lien holder proceeds to enforce it by foreclosure. At that point, the purchaser at the foreclosure sale will hold the property as a tenant in common with the other tenant, but will still have an undivided one-half interest in the property unless and until he brings an action to partition the estate.

O conveys Blackacre to A on Monday. O conveys Blackacre to B on Tuesday. A records on Wednesday. B records on Thursday. If both parties paid valuable consideration for the land, and neither knew of the other's deed, who owns Blackacre?

B, under a notice statute A notice statute is a recording act that alters the common law rule of "first in time, first in right" to protect a subsequent bona fide purchaser ("BFP")- i.e., one who pays valuable consideration and lacks notice of the prior conveyance. A notice statute requires only that the subsequent purchaser have no actual or constructive (i.e., record or inquiry) notice at the time of the conveyance. While a prior grantee can prevent the existence of a subsequent BFP by recording, a BFP will be protected under a notice statute even if she does not record. Here, B prevails over A under a notice statute because B had no notice of the O-A conveyance at the time of her conveyance from O. B would not own Blackacre under a race or race-notice statute (and thus not under every recording act). Under a race statute, the first party to record wins, regardless of whether she has notice of a prior conveyance. Under a race-notice statute, a subsequent BFP prevails over a prior grantee only if she records before the prior grantee. Here, A prevails over B under these acts because B did not record first.

A landowner in fee simple signed a promissory note for $10,000 to a bank, and secured the note by a mortgage of her land to the bank. The mortgage was duly recorded. The landowner then sold the property to an attorney, who assumed and agreed to pay the mortgage to the bank on the land. The attorney did not make payments on the mortgage note to the bank. The bank, following appropriate statutory procedures, foreclosed the mortgage and gave notice to both the landowner and the attorney that it intended to sue for any deficiency. At the foreclosure sale, the property sold for $6,000. The bank now sues both the landowner and the attorney for $5,000, which is the remaining amount of the unpaid principal and interest on the note plus costs of foreclosure. Against which party will the bank be successful in obtaining a judgment?`

Both the landowner and the attorney, although it may only collect once When a grantee assumes the mortgage, the grantee expressly promises the grantor-mortgagor that he will pay the mortgage obligation as it becomes due. The mortgagee then becomes a third-party beneficiary of the grantee's promise to pay and can sue the grantee directly if the grantee fails to pay. After the assumption, the grantor-mortgagor becomes a surety who is secondarily liable to the mortgagee on the note if the grantee fails to pay. The landowner and the attorney are jointly liable, even though the attorney is primarily liable and the landowner is secondarily liable as a surety.

How will the proceeds from a partition sale of property initially held by four joint tenants (A, B, C, and D) be divided if A sold her interest to E, and B died, leaving her property to F and G?

C and D get 3/8 each; E gets 1/4 The distinguishing feature of a joint tenancy is the right of survivorship. When property is held by three or more joint tenants, one joint tenant's conveyance destroys the joint tenancy only as to that interest. The remaining joint tenants continue to hold in joint tenancy as between themselves, and the grantee holds his interest as a tenant in common with them. When A sold her interest to E, that 1/4 interest was severed and thus converted into a tenancy in common, which E continues to hold. Thus, E gets A's 1/4 share. When one joint tenant dies, the property is freed from her interest, and the survivors retain an undivided right in the property. Since B's interest was extinguished on her death, B's devisees do not take B's interest; the surviving joint tenants hold free of it. This leaves C and D as joint tenants with right of survivorship, together owning a 3/4 interest in the land. A joint tenancy is terminated by a suit for partition. When the partition sale was ordered, this joint tenancy was converted into a tenancy in common, and split equally between C and D. Thus, C and D each will receive 3/8 of the partition proceeds.

When property is held in joint tenancy or tenancy in common, which of the following is not a co-tenant's right?

Compel contribution for the cost of improvements Although a joint tenant or tenant in common may have a right to compel contribution from other co-tenants for the cost of necessary repairs, taxes, and payments due on mortgages, she does not have a right to compel contribution for the cost of improvements. Under the unity of possession, each co-tenant has a right to possess the entire estate subject to the equal right of her co-tenant. A co-tenant out of possession cannot bring a possessory action unless there has been an "ouster" (i.e., wrongful exclusion) by the co-tenant in possession. Although a co-tenant generally is not entitled to share in the rental value of the land, she does have a right to share in rents paid by third parties . A joint tenant or tenant in common may mortgage her interest. However, she may not encumber another co-tenant's interest. Note that an individual spouse may not mortgage her interest in tenancy by the entirety property.

Which of the following acts will terminate an easement? response - incorrect Press Enter or Space to submit the answer

Condemnation of the servant estate. The easement holder may be entitled to compensation for the value lost. Use of the easement beyond its legal scope will not terminate an easement. Instead, the easement is surcharged, and the servient owner may sue to enjoin the use. Nonuse of the easement for the statutory period will not terminate an easement. An easement can be extinguished by the easement holder's physical act of abandonment (e.g., erection of a permanent structure over the easement). However, mere nonuse, even for a long period of time, is insufficient to constitute an abandonment of the easement. To terminate the easement, the nonuse must be combined with other evidence of intent to abandon it. Voluntary destruction of the servient estate (e.g., tearing down a building to erect a new one) will not terminate an easement. On the other hand, involuntary destruction of the servient estate (e.g., by fire or flood) will extinguish the easement.

A landlord leased a building to a baker for 10 years, commencing January 1, at a monthly rental of $1,700. The lease stated in part, "The tenant may not sublet or assign this lease without first receiving written permission from the landlord to do so. Any attempt to sublet or assign the lease without first receiving written permission shall constitute a breach entitling the landlord to terminate this lease." Five years later, an investor approached the baker and offered to purchase the bakery if the baker would agree to sublet the premises to him. The baker agreed and executed a sublease on July 1 of that year. The investor took possession the same day. On July 3, the baker approached the landlord and asked for written permission to sublet the premises to the investor. The landlord said he had no real objection to the sublease and would execute the document requested by the baker, but only if the investor would sign a five-year extension of the existing lease. The investor refused to extend the lease, but remained in possession of the building. At no time did the landlord accept rent from the investor. After notice was given to all parties and the applicable grace period in the lease had elapsed, the landlord brought an appropriate action against the baker and the investor to evict them from the premises and to declare the lease terminated because it had been breached. How should the court rule in this action?

For the landlord, because the baker has breached the lease. Generally, if a tenant transfers (assigns or sublets) in violation of a prohibition in the lease against transfers, the transfer is not void. However, the landlord usually may terminate the lease under either the lease terms or a statute. Here, because the baker has breached the provision of the lease prohibiting assignment or sublease, and the lease contains a forfeiture clause, the landlord was within his rights to terminate the lease.

A tenant rented an apartment in a large multi-unit building. One day vandals broke into several of the building's apartments, including the tenant's, and smeared excrement into the carpets and on the walls, and broke out all of the windows. The jurisdiction provides by statute that if a tenant notifies her landlord in writing of a repair that is needed to keep the premises in a habitable condition and the landlord does not repair it within 15 days, the tenant may, at her option, either repair it herself and withhold the expenses from rent, or consider herself constructively evicted and terminate her tenancy. The tenant wrote a letter to the landlord informing him that her walls, carpets, and windows had been damaged and needed repair, and after eight days she received a letter in reply stating that such damages to her apartment were her responsibility to repair. After waiting another week, the tenant paid to have her carpets and walls cleaned and to have her windows replaced. She then withheld the entire next month's rent of $400, because the cleaning and repair bills had totaled $750. After sending her the required statutory notices, the landlord commenced unlawful detainer litigation, seeking to have the tenant evicted for nonpayment of rent. How should the court rule?

For the tenant, because she satisfied the requirements of the statute. Today, however, a majority of jurisdictions, usually by statute, provide for an implied warranty of habitability for residential tenancies. The statute in this question allows the tenant to make the repairs and withhold the cost of the repairs 15 days after notifying the landlord in writing. The statute is applicable because the damage done by the vandals makes the apartment unfit for habitation under whatever standard the court would apply. The tenant has complied with the terms of the statute; she therefore cannot be evicted for nonpayment of rent.

Which of the following does not charge a purchaser of realty with inquiry notice?

His grantor's use of a quitclaim deed. Inquiry notice means that a subsequent grantee is held to have knowledge of any facts that a reasonable inquiry would have revealed, even if he made no inquiry. A quitclaim deed releases whatever interest a grantor might have in the property and contains no covenants for title. Nonetheless, in the majority of states, grantees are not charged with inquiry notice from the mere fact that a quitclaim deed was used.

A developer created an exclusive residential subdivision. In his deed to each lot, the following language appeared: Grantee agrees for himself and assigns to use this property solely as a single-family residence, to pay monthly fees as levied by the homeowners' association for upkeep and security guard services, and that the backyard of this property shall remain unfenced so that bicycle paths and walkways may run through each backyard, as per the subdivision master plan [adequately described], for use by all residents of the subdivision. The developer sold lots to an actuary, a baker, and a coroner. All deeds were recorded. The subdivision was developed without backyard fences, with bicycle paths and walkways in place in accordance with the general plan. The actuary in turn sold to an accountant by a deed that omitted any mention of the covenants above, and the accountant had no actual knowledge thereof. Shortly thereafter, the accountant started operating a tax preparation business out of his home. The baker in turn sold to a barber, who knew of, but refused to pay, the monthly fees levied by the homeowners' association. The coroner leased her property for 10 years to a chiropractor, who erected a fence around the backyard, unaware of the covenant against such fencing. According to common law principles, which of the following statements is correct?

If the barber sues the chiropractor to obtain removal of her backyard fence, the barber would win because the covenant regarding fencing is enforceable in equity against the chiropractor. An equitable servitude is a covenant that, regardless of whether it runs with the land at law, equity will enforce against the assignees of the burdened land who have notice of the covenant. The benefit of an equitable servitude runs to successors if: (i) the original parties so intended, and (ii) the servitude touches and concerns the land. The burden runs if (i) and (ii) are met and (iii) the subsequent purchaser has actual or constructive notice of the covenant. Privity of estate is not needed to enforce an equitable servitude because it is enforced not as an in personam right against the owner of the servient tenement, but as an equitable property interest in the land itself. Here, the original parties intended for the fencing covenant to be enforceable by and against assignees, as shown by the specific language of the covenant ("Grantee agrees for himself and assigns") and its purpose to provide bicycle paths and walkways running through each backyard for the use of all subdivision residents. The benefit of the covenant touches and concerns the barber's property because it increases his enjoyment thereof by providing him with such paths and walkways. Therefore, the barber is entitled to enforce the covenant. The burden of the covenant touches and concerns the land occupied by the chiropractor because it restricts the landholder in her use of the parcel (i.e., her rights in connection with the enjoyment of the land are diminished by being unable to fence in the backyard). The chiropractor will be deemed to have inquiry notice of the restriction because the subdivision is sufficiently developed in accordance with a general plan for the subdivision. Moreover, any neighbor in a subdivision can enforce a covenant contained in a subdivision deed if a general plan existed at the time he purchased his lot. As has been noted, the maintenance of access to all backyards for use as bike paths and walkways was part of such a general plan. Finally, the fact that the chiropractor did not succeed to the coroner's entire estate, but rather a leasehold interest, is irrelevant because privity is not required to enforce an equitable servitude. Therefore, all of the requirements are in place for the existence of an equitable servitude, which can be enforced by the barber against the chiropractor

If L leases property to T, and L subsequently assigns L's interest to L2, whom may T hold liable when X, a paramount title holder, ejects T? response - correct Press Enter or Space to submit the answer

L or L2 A landlord may assign the rents and reversion interest that he owns. The assignee is liable to the tenants for performance of all covenants made by the original landlord in the lease, provided that those covenants run with the land. The original landlord also remains liable on all of the covenants he made in the lease. X's evicting T from the entire leased premises breaches the covenant of quiet enjoyment, which runs with the land. Thus, L and L2 are personally liable to T.

The hostility element of adverse possession requires that the possessor:

Lack the true owner's permission to be on the land It does not mean anger or animosity. Moreover, the state of mind of the adverse possessor is irrelevant-it does not matter whether the possessor believes he is on his own land, knows he is trespassing on someone else's land, or has no idea who owns the land. To establish title by adverse possession, the possessor must show: (i) An actual entry giving exclusive possession that is (ii) Open and notorious, (iii) Adverse (hostile), and (iv) Continuous throughout the statutory period.

If the buyer of land determines that the seller's title is unmarketable, the buyer:

Must notify the seller and give a reasonable time to cure the defects Every land sale contract contains an implied covenant that the seller will provide marketable title at closing. Marketable title is title reasonably free from doubt, which a reasonably prudent buyer would accept. While it need not be perfect title, it must not present the buyer with an unreasonable risk of litigation. Generally, this means an unencumbered fee simple with good record title. If the buyer of land determines that the seller's title is unmarketable, the buyer may NOT sue for damages for breach as soon as the defect is discovered. As stated above, he must notify the seller and give her reasonable time to cure, even if this requires extending the closing date, and even if time is of the essence. If the seller fails to cure the defects, then the buyer may rescind the contract, sue for damages for breach, get specific performance with abatement of the purchase price, or (in some jurisdictions) require the seller to quiet title. Thus, it is not required that the buyer take title to the land "as is." The buyer may NOT sue on the implied covenant of marketable title after closing. This covenant applies at the contract stage of a land sale transaction, before the closing (i.e., exchange of purchase price and deed). The closing extinguishes the contract, which is said to merge with the deed. Then, absent fraud, the seller is no longer liable on this implied covenant; the buyer must rely on any assurances made in the deed.

May a tenant waive the implied warranty of habitability?

No, because such a waiver is against public policy The standard usually applied is the local housing code. One of the reasons for implying a covenant of habitability is to encourage enforcement of the housing code by tenants. Thus, even if the tenant accepts the premises "as is" or covenants to repair, the landlord's obligations under the implied warranty of habitability are usually held to be nonwaivable. The Fair Housing Act bars discrimination based on race, ethnicity, religion, national origin, gender, and disability in the sale or rental of a dwelling. It is not concerned with the implied warranty of habitability.

Fifty-one years ago, an owner conveyed land to a taker for "so long as the land is used solely for residential purposes; otherwise, the interest in land shall revert to the owner and his heirs." The taker used the land as her personal residence for 20 years, but 31 years ago, she began operating a children's day camp on the land. The owner knew of this operation, but he took no action. Two years ago, the aged taker decided to get out of the camp business. She closed her business and once again began to use the land solely as her personal residence. Also two years ago, the owner died, survived by his son and only heir. Now the son is laying claim to the conveyed land. The jurisdiction in which the land is located has a seven-year adverse possession statute and another statute that bars enforcement of possibilities of reverter 55 years after their creation. May the son validly claim title to the land?

No, because the adverse possession period began 31 years ago, and the taker has held the property for more than the requisite seven years. On the happening of the prohibited event (using the land for other than residential purposes), the taker's fee simple determinable automatically came to an end, and the owner was entitled to present possession. Not having claimed possession within the applicable seven-year period, and with the taker's possession being open, notorious, continuous, and adverse, any action by the owner or his heirs is now barred by adverse possession.

A rancher entered into a contract to sell her land to a developer for $60,000. The contract provided that the rancher agreed to convey a good and marketable title to the developer 60 days from the date of the contract. At the time set for closing, the rancher tendered a deed in the form agreed to in the contract. The developer's examination of the record prior to the date of closing disclosed, however, that the owner of record was not the rancher, but a farmer. Further investigation by the developer revealed that, notwithstanding the state of the record, the rancher had been in what the developer concedes is adverse possession for 15 years. The period of time to acquire title by adverse possession in the jurisdiction is 10 years. The developer refuses to pay the purchase price or to take possession because of the "inability" of the rancher to transfer a marketable title. In an appropriate action by the rancher against the developer for specific performance, will the rancher prevail?

No, because the developer cannot be required to buy a lawsuit even if the probability is great that the developer would prevail against the farmer. The seller of land is obligated to deliver a title that is free from reasonable doubt either in fact or law. This does not require a perfect title, but rather one that is free from questions that might present an unreasonable risk of litigation. Title is marketable if a reasonably prudent buyer would accept it in the exercise of ordinary prudence. An inability to establish a record chain of title will generally render the title unmarketable. If the seller attempts to rely on adverse possession to show that defects have been cleared, courts traditionally do not favor such an argument, because proof of adverse possession normally rests on oral evidence, which might not be available to the buyer at a later time. Here, although the rancher may have acquired title by adverse possession, the developer should not be faced with the prospect of having to prove this in court in the future.

Twenty years ago, an uncle conveyed his real property to his niece and nephew as tenants in common. The niece and nephew were estranged, however, so only the niece moved onto the property. Last year, the nephew sued the niece for an accounting for the years that she had exclusive possession of the property. The statutory period for adverse possession in this jurisdiction is 15 years. Is the accounting likely to be granted?

No, because the nephew had the right to use the property but chose not to do so. The niece had the right to possess and enjoy the whole of the property subject to the equal right of the nephew to do the same. The fact that the nephew chose not to exercise his right does not make the niece's possession wrongful. Therefore, an accounting is not warranted here.

A seller owned a two-acre tract of land, on which he built a single-family residence. The seller entered into a contract to sell the land to a buyer for $200,000. One week before closing, the buyer had a survey of the property conducted. It revealed that a portion of the seller's house was 5.98 feet from the sideline. The applicable zoning ordinance requires a six-foot sideline setback. The buyer refused to go ahead with the purchase of the land on the ground that the seller's title was not marketable. If the seller brings suit against the buyer for specific performance, will he prevail?

No, because the seller's title is unmarketable. There is an implied covenant in every land sale contract that at closing the seller will provide the buyer with title that is marketable. It need not be perfect title, but it must be free from questions that might present an unreasonable risk of litigation. Because the placement of the seller's house violated the zoning ordinance, the buyer could be subject to suit.

O conveys a life estate to A, with a remainder to B. If during A's lifetime, X enters into actual, exclusive possession that is open and notorious and hostile for the statutory period, will X obtain title to the land?

No, but X will acquire A's life estate If during A's lifetime, X enters into actual, exclusive possession that is open and notorious and hostile for the statutory period, X will not obtain title to the land, but X will acquire A's life estate. If a landowner does not commence an action to eject a would-be adverse possessor before the statute of limitations expires, she is barred from suing for ejectment, and title vests in the possessor. However, the statute of limitations does not run against the holder of a future interest (e.g., remainder, reversion) until her interest becomes possessory. The future interest holder has no right to possession until the prior present estate terminates, and thus no cause of action for ejectment accrues until that time. Here, X will acquire A's life estate by adverse possession (i.e., a life estate pur autre vie, measured by A's life), but not B's remainder, which remains nonpossessory while A is living. Thus, upon A's death, X's interest will terminate. Land subject to a future interest CAN be acquired by adverse possession; however, the statute will not begin to run as against the future interest holder until her interest becomes possessory, as explained above. This is true regardless of whether X had color of title (i.e., a document purporting to convey title).

A landowner and her neighbor owned adjoining parcels of land. The landowner's property was situated to the west of the neighbor's property. A highway ran along the east of the neighbor's property. Twelve years ago, the landowner asked the neighbor if it would be all right for the landowner to use an eight-foot strip along the northern part of the neighbor's land to access the highway. The only other way for the landowner to get to the highway was to use a one-lane unpaved road that meandered through the woods for two miles. The neighbor agreed, and the landowner used the strip of land regularly to access the highway. The statutory period for adverse possession in this jurisdiction is 10 years. What is the landowner's interest in the neighbor's eight-foot strip of land?

Not an easement. In effect, the landowner only has a "license" (i.e., a revocable privilege) to use the land. The answer is best reached by the process of elimination. Because an easement is an interest in land, the Statute of Frauds applies. Here, the agreement between the landowner and the neighbor was not in writing; thus, the Statute of Frauds requirements for the creation of an express easement were not met.

Which of the following transfers creates an assignment of the lease from T to T2?

One year into a five-year tenancy for years, T transfers his interest "to T2 for four years; however, if T2 breaches the original lease terms, T may reenter and retake the premises". If six months into a tenancy at will, T transfers "my entire interest to T2," the attempted assignment is void and terminates the tenancy at will by operation of law. A tenancy at will is a leasehold estate that is terminable at the will of either the landlord or the tenant. Such a tenancy terminates by operation of law if: 1. Either party dies; 2. The tenant commits waste; 3. The tenant attempts to assign his tenancy; 4. The landlord transfers her interest in the property; or 5. The landlord executes a term lease to a third person. If two years into a four-year tenancy for years, T "assigns my entire interest to T2 for one year," the effect of the transfer is to create a sublease between T and T2. The label given to the transfer by the parties does not determine whether the transfer is an assignment or a sublease. The nature of the transaction is determined by what interest, if any, the tenant retains. Here, although T "assigned" his interest to T2, he transferred only one of the remaining two years of the lease. Thus, the transfer is a sublease rather than an assignment. If four years into a six-year tenancy for years, T orally transfers his entire interest to T2 for two years, the attempted assignment is ineffective under the Statute of Frauds. Most states require that a lease creating a tenancy for more than one year, including an assignment of an interest in a lease for more than one year, be in writing to satisfy the Statute of Frauds.

A landlord leased an apartment to a tenant for five years. The lease provided that the landlord will: (i) keep the apartment building at a comfortable temperature 24 hours per day, and (ii) have the carpets cleaned once a year. Two years later, the landlord began turning off the air conditioning at 10 p.m. The tenant's apartment became hot and stuffy, and she demanded that the landlord honor the covenant. The landlord refused. The following month, the pipes burst in the tenant's only bathroom, rendering it unusable. The resultant flooding soiled some of the carpeting, which had not been cleaned in the past 12 months. The tenant reported the problems to the landlord, who did not return the tenant's phone calls. Which of the following are valid reasons for the tenant to terminate the lease?

Only that the landlord did not fix the bathroom pipes. The general rule at common law was that the landlord was not liable to the tenant for damages caused by the landlord's failure to maintain the premises during the period of the leasehold. Today, however, a majority of jurisdictions, usually by statute, provide for an implied warranty of habitability for residential tenancies. In the absence of a local housing code, the standard applied is whether the conditions are reasonably suitable for human residence. If the landlord breaches the implied warranty, the tenant may: (i) terminate the lease, (ii) make repairs and offset their cost against future rent, (iii) abate rent, or (iv) seek damages. Here, a court is likely to consider the lack of a functioning bathroom as making the premises unsuitable for human residence, allowing the tenant to terminate the lease.

Which statement regarding partial actual eviction is correct?

Partial actual eviction by the landlord relieves the tenant of the obligation to pay rent for the entire premises Every lease contains an implied covenant that neither the landlord nor someone with paramount title will interfere with the tenant's quiet enjoyment and possession of the premises. This covenant is breached by the tenant's total or partial actual eviction from the leased premises. Partial actual eviction occurs when the tenant is excluded from only part of the leased premises. Even though the tenant continues in possession of the remainder of the premises, partial eviction by the landlord relieves the tenant of the obligation to pay rent for the entire premises rather than for only the portion of the premises from which he was evicted. In contrast, partial eviction by a paramount title holder results in an apportionment of rent; i.e., it relieves the tenant of the obligation to pay rent NOT for the entire premises, but only for the portion of the premises from which he was evicted. The tenant remains liable for the reasonable rental value of the portion that he continues to possess.

An owner obtained a loan of $60,000 from a bank in exchange for a promissory note secured by a mortgage on his land, which the bank promptly and properly recorded. A few months later, the owner obtained another loan of $60,000 from a lender, in exchange for a promissory note secured by a mortgage on the land, which the lender promptly and properly recorded. Subsequently, the owner sold the land to a buyer for $150,000 and conveyed a warranty deed. The buyer expressly agreed with the owner to assume both mortgages, with the consent of the bank and the lender. A few years later, the bank loaned the buyer an additional $50,000 in exchange for an increase in the interest rate and principal amount of its mortgage on the land. At that time, the balance on the original loan from the bank was $50,000. Shortly thereafter, the buyer stopped making payments on both mortgages and disappeared. After proper notice to all appropriate parties, the bank instituted a foreclosure action on its mortgage, and purchased the property at the foreclosure sale. At that time the principal balance on the lender's mortgage loan was $50,000. After fees and expenses, the proceeds from the foreclosure sale totaled $80,000. Assuming that the jurisdiction permits deficiency judgments, which of the following statements is most accurate?

The bank keeps $50,000, the lender is entitled to $30,000, and only the lender can proceed personally against the owner for its deficiency. The bank's original mortgage has priority in the proceeds, followed by the lender's mortgage, and only the lender can proceed against the owner because the bank modified its mortgage after the owner had transferred to the buyer. Generally, the priority of a mortgage is determined by the time it was placed on the property, and the proceeds of a foreclosure sale will be used to pay off the mortgages in the order of their priority. However, if the landowner enters into a modification agreement with the senior mortgagee, raising its interest rate or otherwise making the agreement more burdensome, the junior mortgage will be given priority over the modification. Thus, if the first mortgage debt is larger because of the modification, the second mortgage gains priority over the increase in the debt. Here, the bank and the buyer modified the original mortgage by increasing the principal amount and the interest rate. This modification is not given priority over the lender's mortgage, and foreclosure proceeds will not be applied against it because the senior lender's mortgage was not fully satisfied from the proceeds. With regard to the deficiency, the owner is liable to the lender because when a grantee signs an assumption agreement, becoming primarily liable to the lender, the original mortgagor remains secondarily liable on the promissory note as a surety. Here, the buyer assumed the lender's mortgage and became primarily liable; however, the owner remained secondarily liable as surety and can be required to pay off the rest of the lender's mortgage loan. On the other hand, the owner will not be liable to pay off the balance of the bank's loan, because when a mortgagee and an assuming grantee subsequently modify the original obligation, the original mortgagor is completely discharged of liability. The owner had nothing to do with the modification agreed to by the bank and the buyer that increased the amount of the mortgage debt, and will not be even secondarily liable for that amount.

A landowner owned a large parcel of land in a rural area. He built his home on the northern half of the property, and developed a large orchard of fruit trees on the southern portion. A county road ran in front of the northern portion. To service his orchard, the landowner built a driveway directly from the county road across the northern portion of the property to the orchard. To provide electricity to his house, the landowner ran an overhead power line across the orchard property to hook up to the only available electric power pole located on the far southern side of the property. Subsequently, the landowner conveyed the northern parcel to his brother and the southern parcel to his daughter, who said that she did not mind having the power line on the property. Recently, the brother has begun parking his car on the driveway, thus blocking the daughter's access to the southern parcel. Finding no recorded document granting an easement for the power line, the daughter has decided to remove it. If the brother is successful in preventing the daughter from removing the power line, what is the likely reason?

The brother's alternative access to power is much less convenient and would cost 100 times as much. This helps to prove that there was an easement implied by operation of law ("quasi-easement"). An easement may be implied if, prior to the time the tract is divided, a use exists on the "servient part" that is reasonably necessary for the enjoyment of the "dominant part," and a court determines that the parties intended the use to continue after division of the property. To give rise to an easement, a use must be apparent and continuous at the time the tract is divided. In this case, the landowner used the servient part of his property (the southern parcel) to run an overhead power line to the dominant part of his property (the northern parcel). Overhead wires are clearly visible and would be readily discoverable on reasonable inspection. The lines are, therefore, apparent. The use must also be reasonably necessary. Whether a use is reasonably necessary depends on many factors, including the cost and difficulty of the alternatives. This use was reasonably necessary to the enjoyment of the dominant parcel because electricity is important to the enjoyment of the property, and the cost (100 times as much) and difficulty of the alternatives are excessive. Thus, the fact that the use of the southern parcel is reasonably necessary would bolster the brother's case.

A realty company developed a residential development encompassing single-family dwellings, town houses, and high-rise apartments. Included in the deed to each unit was a covenant under which the grantee and the grantee's "heirs and assigns" agreed to purchase electrical power only from a plant that the realty company had constructed within the development. The plant did not supply power outside the development. After constructing and selling half of the units, the realty company sold its interest in the development to an investment firm. The investment firm operated the power plant and constructed and sold the remaining units. Each conveyance from the investment firm contained the same covenant relating to electrical power that the realty company had included in the conveyances it had made. A woman bought a dwelling unit from a man who had purchased it from the realty company. Subsequently, the woman, whose lot was along the boundary of the development, ceased buying electrical power from the investment firm and began purchasing power from a power company that provided such service in the area surrounding the development. The investment firm instituted an appropriate action against the woman to enjoin her from obtaining electrical power from the power company. Both the power company and the investment firm have governmental authorization to provide electrical services to the area. If judgment is for the woman, what is the most likely reason?

The covenant does not touch and concern the land. The investment firm is seeking to enforce the covenant by means of an equitable remedy. Thus, this question concerns an equitable servitude. An equitable servitude relates to a promise that touches and concerns the land. A covenant touches and concerns the land when it makes the land itself more useful or valuable to the benefited party. Here, an agreement to purchase electrical power only from a specified source probably does not touch and concern the land.

To satisfy a debt owed to a creditor, a son executed and delivered to the creditor a warranty deed to a large tract of undeveloped land. The creditor promptly recorded the deed. Shortly thereafter, she built a house on the property and has lived there ever since. The son never actually owned the land. It belonged to his father, but the father had promised to leave the property to the son. Later, the father died and his will devised the property to the son. Pressed for money, the son then sold the land to an investor by warranty deed, which the investor promptly recorded. Although the investor paid full value for the property, he purchased it strictly for investment and never visited the site. He therefore did not realize that the creditor was living there, and knew nothing of the son's earlier deed to the creditor. The jurisdiction in which the land is located has the following statute: "A conveyance of an estate in land (other than a lease for less than one year) shall not be valid against any subsequent purchaser for value without notice thereof unless the conveyance is recorded." Which of the following is the most likely outcome of a quiet title action brought by the creditor against the investor?

The creditor prevails, because the investor was not a purchaser for value without notice of the creditor's interest. When a grantor purports to convey property that he does not own, his subsequent acquisition of title to that property vests in the grantee under the doctrine of estoppel by deed. Most courts, however, hold that this is personal estoppel, which means that title inures to the grantee's benefit only as against the grantor, not a subsequent bona fide purchaser. If the grantor transfers his after-acquired title to an innocent purchaser for value, the bona fide purchaser gets good title. There is a split of authority as to whether the original grantee's recordation of the deed imparts sufficient notice to prevent a subsequent purchaser from being a bona fide purchaser, but the majority view is that it does not because it is not in his chain of title. Thus, it is not the fact that the creditor recorded that prevents the investor from being a bona fide purchaser. The fact that the creditor built a home and was living on the property gave the investor constructive notice of her interest. A title search is not complete without an examination of possession. If the possession is unexplained by the record, the subsequent purchaser is charged with knowledge of whatever an inspection of the property would have disclosed and anything that would have been disclosed by inquiring of the possessor. Therefore, the investor is charged with knowledge of the creditor's possession and with what the creditor would have told him about her possession; i.e., that the property was conveyed to her by the son prior to his conveyance to the investor.

If an easement is said to be surcharged, this means:

The easement's legal scope was exceeded The holder of an easement has the right to use another's land (i.e., the servient tenement), but has no right to possess the land. The scope of an easement is determined by the reasonable intent of the original parties, and when the scope has been specified, these specifics will govern. However, when an easement's scope has been set out only in general language, courts will interpret it to accommodate the holder's present and future reasonable needs. In either event, if the easement holder uses the easement in a way that exceeds its legal scope, the easement is surcharged. The servient landowner may enjoin the excess use and possibly sue for damages if the land has been harmed. However, the easement does NOT terminate by operation of law, nor does such use give the servient owner a power of termination.

Which correctly states the order of priority for allocating mortgage foreclosure sale proceeds, from first to last?

The foreclosing party, any junior lienors in the order of their priority, and then the mortgagor The order of priority for allocating mortgage foreclosure sale proceeds is as follows, from first to last: 1. Expenses of the sale, including attorneys' fees, and court costs; 2. The principal and accrued interest on the foreclosing party's loan; 3. Any junior lienors in the order of their priority; and then 4. The mortgagor. In many cases, no surplus remains after the principal debt is paid off. Senior lienors receive none of the proceeds. Because a senior lien remains on the property (i.e., may itself be foreclosed in the future), a senior lienor is not entitled to any of the money from the sale, even if there is a surplus.

A landowner gratuitously conveyed his interest in land to a friend by quitclaim deed. The friend promptly and properly recorded her deed. Six months later, the landowner conveyed his interest in the same land to an investor for $50,000 by warranty deed, which was promptly and properly recorded. As between the friend and the investor, who has the superior right to title to the land?

The friend, regardless of the type of recording statute. Because the friend recorded prior to the subsequent conveyance, she has the superior right to title regardless of the type of recording statute. A conveyance that is recorded can never be divested by a subsequent conveyance through operation of the recording statutes. By recording, the grantee gives constructive (or "record") notice to everyone. Hence, proper recording prevents anyone from becoming a subsequent bona fide purchaser ("BFP"). Because the landowner's conveyance to the friend was recorded at the time of the landowner's conveyance to the investor, the investor cannot prevail. The investor will clearly lose under a pure race statute because the friend recorded first. The investor will also lose under notice and race-notice statutes because the conveyance to the friend was recorded at the time of the conveyance to the investor. The investor, therefore, had record notice and cannot claim the protection that these types of statutes provide for subsequent purchasers for value who take without notice. The fact that the friend is merely a donee rather than a BFP does not mean that her recording has no effect. It is only the subsequent taker who has to be a BFP rather than a donee to utilize the recording statute. The prior grantee, regardless of her status, protects her interest by recording because it prevents anyone from becoming a subsequent BFP.

A landlord rented an art studio to an artist. Under the terms of the signed, written, two-year lease, the artist agreed to pay the landlord $1,000 per month and to assume responsibility for all necessary repairs. After the first year of the lease, the artist assigned the balance of his lease to a sculptor. The landlord approved the sculptor as a tenant and accepted two rent payments from her, and then the landlord sold the building to an investor. The sculptor had made two payments to the investor when an electrical fire broke out in the studio, injuring the sculptor. The fire was caused by faulty wiring. The landlord was aware that there was a dangerous wiring problem when he leased the property to the artist. But when the landlord discovered how costly repairs would be, he decided it would be more profitable to sell the property than to repair it. The problem was not easily discoverable by anyone other than an expert electrician, and the landlord did not tell the artist, the sculptor, or the investor about the problem. The sculptor sues to recover damages for her injuries. From whom can the sculptor recover?

The landlord, because he failed to disclose a latent defect. If, at the time the lease is entered into, the landlord knows of a dangerous condition that the tenant could not discover upon reasonable inspection, the landlord has a duty to disclose the dangerous condition. Failure to disclose the information about the condition results in liability for any injury resulting from the condition. Because the landlord knew of the dangerous electrical problem at the time he leased the premises to the artist and did not disclose it to either the artist or the sculptor, he is liable for any injuries resulting from that condition.

A retiree purchased a rustic cabin on a small plot of land near the center of a landowner's large parcel of land. The deed to the land, which the landowner delivered to the retiree for fair consideration, did not specifically grant an easement over the landowner's property to reach the public highway bordering her land. There were two means of access to the cabin from the public roads: a driveway from the county road on the south, and a private road from the highway on the east. The landowner told the retiree that he could use the private road from the highway. Twice during his first two years at the cabin, the retiree took the driveway from the county road instead; at all other times he used the private road. At the end of his second year at the cabin, the retiree began reading tarot cards to supplement his retirement income. He had a steady stream of clients coming to his home at all hours of the day and night. Most of the clients came in on the driveway from the county road, which ran close to the landowner's home. The landowner objected, and told the retiree that neither he nor his clients had any right to use that driveway and that they must use the private road from the highway. The retiree refused, and he and his clients continued to use the driveway from the county road for three years. Finally, the landowner began blocking off the driveway from the county road. The retiree brought suit to enjoin this practice. The prescriptive period in this jurisdiction is five years. Who will most likely prevail?

The landowner, because she may select the location of the easement. The landowner will prevail in a suit because she, as the holder of the servient estate, has the right to choose the location of an easement by necessity. An easement by necessity arises when the owner of a tract of land sells a part of the tract and by this division deprives one lot of access to a public road or utility line. The owner of the servient parcel has the right to locate the easement, provided the location is reasonably convenient. The landowner has chosen the private road from the highway; thus, the retiree has no right to use the driveway from the county road.

A farmer conveyed a 60-acre parcel of land to a rancher. A private gravel road ran through the center of the parcel. The southern half consisted of arable land, which the farmer, and later the rancher, used for farming. The northern half was undeveloped woodland. The rancher never used the northern half for timbering or for anything else. On very rare occasions, the rancher would take a walk in the woods, but outside of those occasions she never set foot on the northern half. Fifteen years after the farmer conveyed the parcel to the rancher, a landowner appeared, claiming ownership of the northern half of the parcel. Unbeknownst to either the farmer or the rancher, the landowner's name had been forged on the deed purporting to convey the parcel to the farmer, and the landowner was, in fact, the true owner of the property at that time. The state in which the parcel is located has a 10-year statutory adverse possession period. The landowner admits that the rancher now has title to the southern half of the parcel by adverse possession. In an action to quiet title, who will prevail as to the northern half of the parcel?

The landowner, because the rancher did not actually occupy the northern half. An adverse possessor will gain title only to the land she actually occupies. Actual possession is the kind of use the true owner would make of the parcel and is designed to give the owner notice of the trespass and the extent of the adverse possessor's claim. The gravel road divides the parcel into two distinct lots, and the rancher's use of the northern half was not sufficient to put the landowner on notice of her trespass.

A developer prepared and recorded a subdivision plan, calling for 100 home sites on half-acre lots. There were five different approved plans from which a purchaser could choose the design of the home to be built on his lot. Each deed, which referred to the recorded plan, stated that "no residence shall be erected on any lot that has not been approved by the homeowners' association." A lawyer purchased a lot and built a home based on one of the approved designs. However, many of the lots were purchased by investors who wanted to hold the lots for investment purposes. Two years after the lots went on the market, one such investor sold her lot to an architect by a deed that did not contain any reference to the recorded plan nor the obligation regarding approval by the homeowners' association. In fact, because very few residences had been built in the subdivision since the lots were first available for purchase, no homeowners' association meetings had been held in two years. The architect began building a very modernistic house on her one-half acre. When the lawyer noticed the house being built, he brought an action to enjoin the construction. For which party will the court rule?

The lawyer, because the recorded subdivision plan, taken with the fact that all lots were similarly restricted and the architect had notice of this, gave him the right to enforce the covenant on her property. When a subdivision is created with similar covenants in all deeds, there is a mutual right of endorsement (each lot owner can enforce against every other lot owner) if two things are satisfied: (i) a common scheme for development existed at the time that sales of parcels in the subdivision began; and (ii) there was notice of the existence of the covenant to the party sued. Here, there was a common scheme evidenced by the recorded plan, and the fact that the covenant was in the architect's chain of title gave her constructive notice of the restriction. Therefore, not only does the covenant apply to the architect's land, but the lawyer (or any other lot owner) can enforce it as a reciprocal negative servitude.

Twenty-five years ago, a man purchased a vacant tract of land from a woman. Unbeknownst to the man, the woman did not own the land. Someone else owned the land in fee simple. Shortly after the purchase, the man built a house on the northwest quarter of the tract, leaving the rest of the tract vacant. Recently, the actual owner of the tract died, still without knowledge that the man had built a house on the northwest corner of the tract. The actual owner's will left all of his property to his son. The relevant statutory period for adverse possession is 20 years. If the man brings suit to quiet title to the tract he had purchased 25 years ago, how should the court decide?

The man is the owner of the entire tract. The man would be declared the owner of the tract on the basis of constructive adverse possession. To establish title by adverse possession, the possessor must show (i) an actual entry giving exclusive possession that is (ii) open and notorious, (iii) adverse (hostile), and (iv) continuous throughout the statutory period. Here, the man actually possessed at least a quarter of the property because he built a house on it. The man used the house alone, so his possession was exclusive. A house is plainly visible, so the possession was open and notorious. The man built the house without the true owner's permission, so the possession was hostile. And the possession was continuous for 25 years, longer than the statutory period. Actual possession of a portion of a unitary tract of land is sufficient adverse possession as to give title to the whole of the tract of land after the statutory period, as long as there is a reasonable proportion between the portion actually possessed and the whole of the unitary tract, and the possessor has color of title to the whole tract. Color of title is a document that purports to give title, but for reasons not apparent from its face does not. Usually, the proportion will be held reasonable if possession of the portion was sufficient to put the owner or community on notice of the fact of possession. Here, the man had color of title to the entire tract because he purportedly purchased it from the woman. His house took up a significant portion of the property, such that the owner or community would have been on notice of the man's possession of the tract.

A man owned a tract of land in fee simple. Fifteen years ago, he built a barn on five acres that he believed were part of his property. One year later, the man discovered that the five acres on which he had built his barn were not part of his property. The five acres actually belonged to the woman who owned the adjoining property. The year following the discovery that the five acres belonged to the woman next door, the woman died, leaving all of her property to her one-year-old daughter. The man has brought a quiet title action against the now 14-year-old daughter. The statutory period for adverse possession in this jurisdiction is 10 years. The man has not paid any additional property taxes to account for the five acres for any of the past fifteen years. Who will prevail?

The man, because he was in continuous possession of the five acres for the statutory period. To establish title by adverse possession, the possessor must show (i) an actual entry giving exclusive possession that is (ii) open and notorious, (iii) adverse (hostile), and (iv) continuous throughout the statutory period. Here, the man possessed the property by building a barn on it, something that is clearly visible to the public, so the possession was actual and open and notorious. The man did not share the barn with anyone, so the possession was exclusive. The man did not have the true owner's permission to build the barn, so the possession was hostile. And the possession was continuous for more than 10 years.

Which of the following is required for the burden of an equitable servitude to run to a subsequent purchaser of the land?

The purchaser must have notice of the covenant. An equitable servitude is a covenant (i.e., a promise to do or not do something on the land) that, regardless of whether it runs with the land at law, can be enforced in equity against assignees of the burdened land who have notice of it. The burden of an equitable servitude will run to a subsequent purchaser if: 1. The covenanting parties intended that successors in interest be bound by the covenant; 2. The purchaser has notice of the covenant; and 3. The covenant touches and concerns the land (i.e., it benefits the covenantor and his successor in their use and enjoyment of the burdened land). The requisite notice may be acquired through actual notice (direct knowledge of the covenants in the prior deeds); inquiry notice (the neighborhood appears to conform to common restrictions); or record notice (if the prior deeds are in the grantee's chain of title he will, under the recording acts, have constructive notice of their contents). Thus, there the restriction need not be in the buyer's record chain of title for the buyer to be burdened by it-as long as the buyer has some kind of notice. Horizontal privity between the original covenanting parties is not required. Horizontal privity means the original parties to a real covenant shared some interest in the land independent of the covenant at the time they entered it (e.g., as grantor and grantee). Horizontal privity is required to enforce the burden of a real covenant at law, but it is not required to enforce the burden of an equitable servitude. A common scheme for development is not required for the burden of a written equitable servitude to run to a subsequent purchaser. Generally, equitable servitudes are created by covenants contained in a writing that satisfies the Statute of Frauds. However, reciprocal negative servitudes may be implied absent a writing if there is a common scheme for the development of a subdivision and the grantee had actual, record, or inquiry notice of restrictions that do not appear in his deed. The common scheme exception applies only to negative covenants and equitable servitudes; affirmative covenants must be in writing.

Two partners bought a commercial building from an owner. They paid cash for the building and took title as joint tenants with right of survivorship. Several years later, the first partner executed a mortgage on the building to secure a personal loan to a bank. The second partner had no knowledge of the mortgage to the bank. The state in which the commercial building is located recognizes the lien theory of mortgages. The first partner died before paying off his loan. He left all of his property by will to his daughter, his only heir. Who has title to the commercial building?

The second partner has title free and clear of the mortgage. When the partners bought the property, they took title as joint tenants with right of survivorship. If the joint tenancy continued until the first partner's death, then the property would pass immediately on death to the second partner. Because the second partner did not sign the mortgage, she would not be subject to it, regardless of whether she knew about it. The key to answering this question is to know whether execution of the mortgage by the first partner caused a severance of the joint tenancy. If it did cause a severance, then the first partner's one-half would not pass to the second under right of survivorship but instead would pass to the first's estate, and thus would go to the daughter by will. Whether a mortgage creates a severance or not depends on whether the state follows the lien theory or the title theory of mortgages. Lien theory means no severance; title theory means severance. Because this is a lien theory state (majority rule on the MBE), there was no severance; thus, the joint tenancy remained intact. On the first partner's death, the joint tenancy ended and the first partner's interest instantly passed to the second partner. The first partner's estate got nothing; hence, the daughter could get nothing.

On April 15, a seller entered into a valid written agreement to sell her home to a buyer for $175,000. The provisions of the agreement provided that closing would be at the buyer's attorney's office on May 15, and that the seller would deliver to the buyer marketable title, free and clear of all encumbrances. On the date of closing, the seller offered to the buyer the deed to the house, but the buyer refused to go ahead with the purchase because his attorney told him that a contractor who had done work on the house had recorded a lis pendens on May 1 against the property regarding a $10,000 contract dispute he had with the seller. The seller indicated that she was unaware of the lien, but that she was willing to go ahead with the sale and set aside funds from the purchase price to cover the contractor's claim until the dispute was resolved. The buyer still refused to proceed, stating that the seller had breached the contract. If the seller brings an action against the buyer for specific performance, what is the probable result?

The seller prevails, because an implied term of their contract was that she could use the proceeds to clear any encumbrance on the title. In a contract for the sale of real property, the seller of the land is entitled to use the proceeds of the sale to clear title if she can ensure that the purchaser will be protected. The seller's offer to escrow the funds in this case should act as such guarantee.

A buyer purchased a parcel of land from a seller for $500,000. The buyer financed the purchase by obtaining a loan from the seller for $300,000 in exchange for a mortgage on the land. The seller promptly and properly recorded his mortgage. Shortly thereafter, the buyer gave a mortgage on the land to a creditor to satisfy a preexisting debt of $100,000 owed to the creditor. The creditor also promptly and properly recorded its mortgage. Within a year, the buyer stopped making payments on both mortgages, and the seller brought an action to foreclose on his mortgage. The creditor was not included as a party to the foreclosure action. The seller purchased the property at a public foreclosure sale in satisfaction of the loan. The creditor subsequently discovered the sale and informed the seller that it was not valid. Who has title to the land?

The seller, but he must redeem the creditor's mortgage to avoid foreclosure. As a general rule, the priority of a mortgage is determined by the time it was placed on the property. When a mortgage is foreclosed, the purchaser at the sale will take title as it existed when the mortgage was placed on the property. Thus, foreclosure will terminate interests junior to the mortgage being foreclosed but will not affect senior interests. However, if a lien senior to that of a mortgagee is in default, the junior mortgagee has the right to pay it off (i.e., redeem it) to avoid being wiped out by its foreclosure. Thus, those persons with interests subordinate to those of the foreclosing party are necessary parties to the foreclosure action. Failure to include a necessary party results in the preservation of that party's interest despite foreclosure and sale. Hence, the seller's failure to include the creditor as a party to the foreclosure action preserved the creditor's mortgage on the property. To avoid the creditor's foreclosing (because the buyer was in default of the creditor's mortgage as well), the seller will need to pay off the creditor's mortgage.

A man and a woman purchased a parcel of land, taking title as joint tenants. Two years later, they married and had a son. Several years after that, the man and woman divorced. After the divorce, the woman and her son continued to occupy the land, although title remained in the names of both the man and the woman. The man moved out of the state and conveyed all of his title and interest in the land by deed to the son. Shortly thereafter, the man was killed in an automobile collision. The man died intestate. Who has title to the land?

The woman and her son as tenants in common. An inter vivos conveyance by one joint tenant of his undivided interest severs the joint tenancy, so that the transferee takes the interest as a tenant in common and not as a joint tenant. Here, there was an inter vivos conveyance by the man to the son of all of the man's interest in the property held in joint tenancy with the woman. This conveyance destroyed the joint tenancy, so that the son takes his interest in the property as a tenant in common with the woman, rather than as a joint tenant.

Which of the following transfers creates a sublease from T to T2?

Two years into a four-year tenancy for years, T "assigns my entire interest to T2 for one year" The label given by the parties does not determine whether a transfer is an assignment or a sublease. Rather, a complete transfer of a tenant's entire remaining lease term is an assignment, and a transfer retaining any part thereof is a sublease. Here, although T "assigned" his interest to T2, he transferred only one of the remaining two years of the lease. Thus, the transfer is a sublease rather than an assignment.

A man had rented a woman's home from her for seven years. When the time came to sign a new lease, the woman decided that because the man had always been a quiet tenant, she would continue to charge him only $350 per month rent instead of the $500 to $550 she could probably get otherwise. The new lease was for a period of five years, and by its terms, the man was specifically prohibited from assigning the lease without the woman's specific written consent. About a year later, the man got married and moved into his new wife's home. Instead of giving up his lease, the man sublet the property to a friend for $500 a month. The man did not get the woman's permission to sublease the property. If the woman brings an action to either eject the friend from the premises or to recover damages from the man for subletting the premises without her consent, what is the most likely result?

The woman will have no cause of action for either ejectment or damages. There are two ways for a tenant to transfer the right to possession under a lease: assignment (transferring the entire period of time remaining under the lease) and sublease (transferring only a portion of the time remaining under the lease). Restraints on alienation are traditionally strictly construed. Thus, a covenant prohibiting assignment does not prohibit subleasing and vice versa. Hence, this prohibition against assignment would not be read to include a prohibition against subleasing. Therefore, the woman would have no cause of action against the man

Which of the following is not required for the burden of an equitable servitude to run to successors in interest?

There is vertical privity between the covenantor and his successor in interest. ertical privity between the covenantor and his successor in interest is not required for the burden of an equitable servitude to run to successors in interest. An equitable servitude is a covenant (i.e., a promise to do or not to do something on the land) that, regardless of whether it runs with the land at law, can be enforced in equity against assignees of the burdened land who have notice of the covenant. The burden of an equitable servitude will run to successors in interest if: 1. The covenanting parties intended that successors in interest be bound by the covenant; 2. The successor in interest has notice of the covenant (if she has given value); and 3. The covenant touches and concerns the land (i.e., it benefits the covenantor and his successor in their use and enjoyment of the burdened land). Horizontal privity between the original covenanting parties and vertical privity between the covenantor and his successor in interest are not required.

If a mortgage exists on property when a real estate contract is signed:

Title may be marketable Every land sale contract contains an implied covenant that the seller will furnish marketable title on the date of closing. Generally, encumbrances (i.e., mortgages, liens, easements, and covenants) render title unmarketable. However, a seller has the right to satisfy a mortgage or lien at the closing with sale proceeds. Thus, if the purchase price is sufficient and this is accomplished simultaneously with the transfer of title, the buyer cannot claim that the seller's title is unmarketable.

If a landlord's breach of duty renders the premises unsuitable for occupancy, under the doctrine of constructive eviction, the tenant may:

Vacate the premises, terminate the lease, and sue for damages Under the doctrine of constructive eviction, if the landlord's breach (i.e., doing an act or failing to provide some service that he has a legal duty to provide) makes the premises untenantable, the tenant may terminate the lease and also may seek damages if the following conditions are met: 1. The breach must be by the landlord or by persons acting for him. 2. The breach must substantially and materially deprive the tenant of her use and enjoyment of the premises (e.g., flooding, absence of heat in winter). 3. The tenant must give the landlord notice and a reasonable time to repair. 4. The tenant must vacate the premises within a reasonable time. Because a tenant cannot claim a constructive eviction unless and until she vacates the premises, her remedies do not include remaining in possession of the premises and refusing to pay rent until the interference ceases or continuing to pay rent and suing for damages. The tenant is not limited to suing for breach only if the lease contained an express covenant for quiet enjoyment. Every lease contains an implied covenant that neither the landlord nor someone with paramount title will interfere with the tenant's quiet enjoyment and possession of the premises. If a landlord does so, the tenant has the remedies discussed above.

Must a junior mortgagee be named as a party to a senior mortgagee's foreclosure action?

Yes, because it has the right to pay off the senior mortgage to avoid being wiped out by foreclosure Foreclosure destroys interests ( e.g., liens, mortgages, leases, easements) junior to the mortgage being foreclosed. Thus, if a senior mortgage is in default, a junior mortgagee has the right to pay it off (i.e., redeem it) to avoid being wiped out by its foreclosure. Failure to join the junior mortgagee results in the preservation of its interest despite foreclosure and sale. In contrast, those with interests senior to that of the foreclosing party are not necessary parties because their interests are not affected by foreclosure. The buyer at the sale takes subject to senior interests, which remain on the land. As explained above, NOT all those with liens on the property are necessary parties to a foreclosure action. Only those with interests subordinate to that of the foreclosing party must be named in the foreclosure action. Failure to name a senior interest holder does not affect that party's interest. Because foreclosure extinguishes interests junior to the mortgage being foreclosed, as explained above, junior mortgagees are necessary parties to a senior mortgagee's foreclosure action. Foreclosure DOES affect interests junior to the mortgage being foreclosed. As explained above, junior mortgages are extinguished by foreclosure so long as they are joined in the action.

A buyer entered into a written contract with a seller to purchase his commercial property for $100,000. The contract did not specify the quality of title to be conveyed, and made no mention of easements or reservations. The closing was set for November 25, three months from the signing of the contract. Shortly thereafter, the buyer obtained a survey of the property, which revealed that the city had an easement for the public sidewalk that ran in front of the store. Because this actually enhanced the value of the property, the buyer did not mention it to the seller. Subsequently, the buyer found a better location for her business. On November 1, the buyer notified the seller that she no longer intended to purchase the property. The seller told her that he intended to hold her to her contract. At closing, the buyer refused to tender the purchase price, claiming that the seller's title is unmarketable and citing the sidewalk easement as proof of that fact. In a suit for specific performance, will the seller likely prevail?

Yes, because the buyer was aware of the visible easement and it enhanced the value of the property. The seller will prevail in his suit for specific performance because the easement was visible, the buyer was aware of it at the time she entered into the contract (i.e., she knew a public sidewalk ran in front of the store), and the easement enhanced the value of the property. There is an implied covenant in every land sale contract that, at closing, the seller will provide the buyer with marketable title. Marketable title is title reasonably free from doubt, which generally means free from encumbrances and with good record title. Easements are generally considered encumbrances that render title unmarketable; so if an easement is not provided for in the contract, it usually renders the seller's title unmarketable. There is an exception, however. A majority of courts have held that a beneficial easement that was visible or known to the buyer does not constitute an encumbrance. In this case, the sidewalk was visible, known to the buyer, and beneficial to the property. Thus, the sidewalk easement does not impair the marketability of the seller's title. Therefore, the buyer's excuse for her nonperformance is not valid, and because land is involved, the seller can get specific performance of the contract for purchase of the property.

A homeowner leased his home to a tenant for three years. The following year, the homeowner conveyed the house to a buyer, who never recorded her deed nor did anything with regard to the house. The tenant continued paying rent to the homeowner. Three months after the conveyance to the buyer, the homeowner conveyed the property to his cardiologist, who knew nothing of the prior conveyance to the buyer. The homeowner took the cardiologist's money and skipped town. The cardiologist told the tenant that he now owned the house and that all rents should be paid to him. The tenant complied. Six months later, the cardiologist went to his local bank for a loan. He offered to put up the property as security. The bank discovered that the cardiologist had never recorded his deed and that, just two weeks prior to his loan application, the buyer had recorded a deed to the house that bore an earlier date than the deed the cardiologist had shown the bank. Because of this cloud on the title, the bank refused the loan request. When the tenant discovered this, she quit paying rent to the cardiologist. The state has a recording statute that provides, "a conveyance of an interest in land, other than a lease for less than one year, shall not be valid against any subsequent purchaser for value, without notice thereof, unless the conveyance is recorded." If the cardiologist sues the tenant to compel the payment of rent, is the cardiologist likely to win?

Yes, because the cardiologist was a bona fide purchaser when he bought the property from the homeowner. When the cardiologist bought the property from the homeowner, he was a bona fide purchaser who gave value and who had no notice of the earlier sale to the buyer. Not only did the cardiologist not have actual notice of the earlier sale, he did not have constructive notice either because the buyer did not record before the cardiologist bought. The recording statute in the jurisdiction is a notice statute. In a jurisdiction with a notice recording statute, a subsequent purchaser who gives value and takes without notice wins over the earlier grantee. If the facts had shown a jurisdiction with a race-notice recording act, the cardiologist would have been in trouble. With race-notice, the cardiologist would not only have to take without notice, he would have to be the first to record. Because the facts do not show that the cardiologist recorded at all, he would lose. But because this is a notice act jurisdiction, the fact that the buyer finally recorded before the cardiologist is irrelevant.

A landlord leased a house to a tenant for five years. Under the terms of the lease, the tenant was to pay a fixed monthly rent plus all taxes and reasonable maintenance charges for the upkeep of the house. Three years into the lease, the tenant assigned her lease to a friend by written agreement. Although the tenant properly set forth the terms concerning the rent and maintenance charges, she failed to properly state that the friend was liable to pay the taxes on the residence during the period of the lease. A year later, the landlord received notice that a tax lien would be placed on the residence unless the taxes were immediately paid. The landlord paid the taxes and brought suit against the tenant's friend for the amount. The suit extremely upset the friend, who abandoned the residence. Can the landlord successfully bring a suit against the tenant for this breach of the lease?

Yes, because the tenant's assignment to the friend did not terminate the tenant's obligations. An assignee is in privity of estate with the landlord and is liable for all covenants that run with the land, including the covenant to pay rent. The original tenant (assignor) remains in privity of contract with the landlord and is liable for the rent reserved in the lease if the assignee abandons the property. Therefore, the tenant is liable to the landlord for the remaining rent.

A landowner owned a large tract of land, which he divided into two parcels. The northern parcel abutted a public highway. The shortest route from the southern parcel to the highway was over a private road that crossed the northern parcel. The other route was over a single-lane dirt and gravel path that wound for over four miles through the woods. The landowner sold the southern parcel to a developer, including an express easement in the private road across the northern parcel. The landowner knew of the developer's plans to open an inn on the property. The developer built the inn but never opened it to the public. Fifteen years later, the developer sold the southern parcel to an investor, who planned to open the inn to the public. The developer had never properly recorded her deed to the land, but the investor promptly recorded her deed, which made no mention of a right to cross the northern parcel via the private road. About a week after the investor took possession of the southern parcel, she learned of the provision in the developer's deed to the land. However, the landowner refuses to grant the investor permission to use the road across his property to reach the highway. Does the investor have a right to cross the northern parcel?

Yes, even if the developer never exercised her right to use the easement when she owned the southern parcel. The original easement granted to the developer was an easement appurtenant, the benefit of which passes with a transfer of the benefited land. An easement is deemed appurtenant when the right of special use benefits the easement holder in her physical use or enjoyment of another tract of land. The land subject to the easement is the servient tenement, while the land having the benefit of the easement is the dominant tenement. The benefit of an easement appurtenant passes with transfers of the benefited land, regardless of whether the easement is mentioned in the conveyance. All who possess or subsequently succeed to title to the dominant tenement are entitled to the benefit of the easement. The easement granted to the developer was an easement appurtenant because the right to use the private road across the northern parcel (the servient tenement) benefited the developer in her use and enjoyment of the southern parcel (the dominant tenement) by providing her with the most convenient access to the public highway. Thus, when the developer sold the benefited land to the investor, the benefit of the easement also passed to the investor as an incident of possession of the southern parcel.

Is a long period of nonuse sufficient to terminate an easement?

Yes, if it is accompanied by other evidence of intent to abandon the easement. However, a long period of nonuse on its own, even if it continues for the statutory period, is insufficient to constitute abandonment.

In a residential subdivision, will a commercial builder be bound by a residential-use restriction that was omitted from his deed?

Yes, if the builder had inquiry notice of a common scheme for development. An equitable servitude is a covenant (i.e., a promise to do or not do something on the land) that, regardless of whether it runs with the land at law, can be enforced in equity against assignees of the burdened land who have notice of it. Generally, equitable servitudes are created by covenants contained in a writing that satisfies the Statute of Frauds. However, in the absence of a writing, reciprocal negative servitudes may be implied if: 1. There is a common scheme for development (i.e., a plan existing at the time sales of the subdivision parcels began that all parcels be developed within the terms of the negative covenant); and 2. The grantee had actual, record, or inquiry notice of the covenant. Thus, the builder may be bound without actual notice of restrictive covenants in the deeds to other lots. In a residential subdivision, the builder would be on inquiry notice of a common scheme for development if the neighborhood appeared to conform to common restrictions. Thus, the builder would be bound by the residential-use restriction. Even though there is no written restrictive covenant in the deed to the builder's lot, the restriction may be enforced as a reciprocal negative servitude, discussed above. To be bound by the restriction, the builder need NOT be in horizontal privity with the developer. Horizontal privity requires that the original parties to a real covenant shared some interest in the land independent of the covenant at the time they entered it (e.g., as grantor and grantee). Horizontal privity is required to enforce the burden of a real covenant at law, but it is not required to enforce the burden of an equitable servitude.

May a grantee be bound by a covenant that does not appear in his deed or chain of title?

Yes, if there is a common scheme for development and the grantee had notice of the covenant. A grantee may be bound by a covenant that does not appear in his deed or chain of title if there is a common scheme for development and the grantee had notice of the covenant. An equitable servitude is a covenant (i.e., a promise to do or not to do something on the land) that, regardless of whether it runs with the land at law, can be enforced in equity against successors to the burdened land who have notice of the covenant. Generally, equitable servitudes are created by covenants contained in a writing that satisfies the Statute of Frauds. However, in the absence of a writing, negative equitable servitudes may be implied if (i) there is a common scheme for development (i.e., a plan existing at the time sales of the subdivision parcels began that all parcels be developed within the terms of the negative covenant), and (ii) the grantee had actual, record, or inquiry notice of the covenant. Except as explained above, a grantee may not be bound by a covenant that does not appear in his deed or chain of title even if the covenant touches and concerns the land. The burden of an equitable servitude will run to successors in interest if: (i) the covenanting parties intended that successors in interest be bound by the covenant; (ii) the successor in interest has notice of the covenant; and (iii) the covenant touches and concerns the land (i.e., it benefits the covenantor and her successor in their use and enjoyment of the burdened land). In the absence of a writing, however, the servitude will not be enforced unless there is a common scheme for development as explained above. A grantee may be bound by a covenant that does not appear in his deed or chain of title even if the deed contains a covenant against encumbrances. This is a covenant contained in a general warranty deed assuring that there are neither visible encumbrances (e.g., easements) nor invisible encumbrances (e.g., mortgages) against the title or interest conveyed. The presence of this covenant does not affect the ability of a successor in interest to the covenantee to enforce an equitable servitude. A grantee also may be bound by a covenant that does not appear in his deed or chain of title even if the deed is a quitclaim deed. This type of deed conveys whatever interest, if any, the grantor has in the property. It does not affect the ability of a successor in interest to the covenantee to enforce an equitable servitude.

On February 10, an owner took out a $10,000 mortgage on her land with a bank. On February 15, the owner conveyed the land for $50,000 to a buyer who was not aware of the mortgage. On February 17, the bank recorded its mortgage interest in the land. On February 21, the buyer recorded his deed to the land. Does the buyer hold the land subject to the bank's mortgage?

Yes, in a race-notice jurisdiction. All recording acts apply to mortgages as well as deeds. Thus, a subsequent purchaser of the property will take subject to a prior mortgage unless the recording act changes the result. A race-notice recording act would change this result only where a subsequent purchaser did not have notice of the mortgage at the time of purchase and recorded his deed before the mortgage was recorded. Here, the buyer did not have notice of the mortgage but he recorded after the bank; thus, he takes subject to the bank's interest.

If a residential tenant assuming possession of the leased premises discovers that the wiring is dangerously frayed and the plumbing is faulty, the landlord has breached the __________.

implied warranty of habitability Under the implied warranty of habitability for residential tenancies, the landlord covenants that the premises are suitable for human residence. The standard usually applied is the local housing code. Dangerous wiring and faulty plumbing likely violate the local housing code and will subject the landlord to liability for breach of the implied warranty of habitability. The covenant against encumbrances is not a landlord-tenant concept but is a usual covenant contained in a general warranty deed. It is a covenant assuring that there are neither visible encumbrances (e.g., easements) nor invisible encumbrances (e.g., mortgages) against the title or interest conveyed. The implied covenant of marketability is not a landlord-tenant concept but is implied in every land sale contract. It obliges the seller to provide the buyer with marketable title (i.e., title reasonably free from doubt) at closing. Under the covenant of quiet enjoyment, the landlord covenants that neither she nor someone with paramount title will interfere with the tenant's quiet enjoyment and possession of the premises. The covenant of quiet enjoyment may be breached by: total actual eviction, partial actual eviction, or constructive eviction.

A recording act that provides: "Any conveyance of an interest in land, other than a lease for less than one year, shall not be valid against any subsequent purchaser for value, without notice thereof, unless the conveyance is recorded," is a __________.

notice statute Under a notice statute, a later purchaser of land will prevail over an earlier grantee if she takes without actual or constructive (e.g. record) notice of the earlier grant. Example of race-notice statute: any conveyance of an interest in land, other than a lease for 1 yr, shall not be valid against any subsequent purchaser for value, w/o notice thereof, whose conveyance is first recorded." Under race-notice statute, a later purchaser will prevail over an earlier grantee only if she takes without actual or constructive (e.g. record) notice of the earlier grant and records before he does. Pure race statute: "Any conveyance of an interest in land, other than a lease for less than 1 yr, shall not be valid against any subsequent purchaser whose conveyance is first recorded." Notice is irrelevant. First party to record, regardless of date of conveyance, wins.


Conjuntos de estudio relacionados

Real Bis: VA Law and Regulations - Chapter 2

View Set

General Biology I - 4.7: The Energy-Related Organelles

View Set

3.1-3.4 US History Review Questions

View Set

Unit 6: Efficient Market Hypothesis and its Critiques

View Set

A&P I - Ex8: Nervous Tissue & Spinal Cord

View Set

8 th English (4) In the Bazaars of Hyderabad

View Set